A Simple(?) Differential Equation

Click For Summary
The discussion revolves around solving a differential equation related to the motion of an object under the influence of an exponential acceleration. The problem requires finding the distance traveled over time t, starting from rest, with the given acceleration equation. The initial attempt at integration led to incorrect results, prompting clarification on the integration process and the application of initial conditions. Correct integration yields a solution that incorporates constants derived from initial conditions, ultimately aligning with the expected results for small and large t. The exchange emphasizes the importance of properly handling constants of integration in differential equations.
blintaro
Messages
37
Reaction score
1
This is question 1.4 of Chapter 8 of Mary L. Boas's Mathematical Methods in the Physical Sciences, Edition 2. I'm using it as a substitute for my ordinary differential equations class since my textbook has apparently been lost somewhere in the mail.

Homework Statement


Find the distance which an object moves in time t if it starts from rest and has an acceleration d^2/dt^2 = ge^(-kt). Show that for a small t the result is approximately (x = 1/2(gt^2)) and for very large t, the velocity dx/dt is approximately constant. (This problem corresponds roughly to the motion of a parachutist.)


Homework Equations


The solution manual gives the solution as: x=k^(-1)gt + k^(-2)g(e^(-kt)-1)


The Attempt at a Solution


I integrated both sides of d^2/dt^2 = ge^(-kt) twice with respect to t, ending up with x = k^(-2)ge^(-kt) + v_0(t) + x_0.
However substituting t=0 into the equation gives x = g/(k^2) + x_0, not (1/2)gt^2, so, obviously on the wrong track.

I put the process in an attached image below since it might be easier to read that than my terrible equation formatting. Thanks!
 

Attachments

  • Screen shot 2014-07-05 at 1.33.40 PM.png
    Screen shot 2014-07-05 at 1.33.40 PM.png
    120.9 KB · Views: 955
Physics news on Phys.org
blintaro said:
This is question 1.4 of Chapter 8 of Mary L. Boas's Mathematical Methods in the Physical Sciences, Edition 2. I'm using it as a substitute for my ordinary differential equations class since my textbook has apparently been lost somewhere in the mail.

Homework Statement


Find the distance which an object moves in time t if it starts from rest and has an acceleration d^2/dt^2 = ge^(-kt). Show that for a small t the result is approximately (x = 1/2(gt^2)) and for very large t, the velocity dx/dt is approximately constant. (This problem corresponds roughly to the motion of a parachutist.)

Homework Equations


The solution manual gives the solution as: x=k^(-1)gt + k^(-2)g(e^(-kt)-1)

The Attempt at a Solution


I integrated both sides of d^2/dt^2 = ge^(-kt) twice with respect to t, ending up with x = k^(-2)ge^(-kt) + v_0(t) + x_0.
However substituting t=0 into the equation gives x = g/(k^2) + x_0, not (1/2)gt^2, so, obviously on the wrong track.

I put the process in an attached image below since it might be easier to read that than my terrible equation formatting. Thanks!

Please post your equations instead of images of them as per forum rules. Your integrations are not correct. For example, if ##x''(t) = ge^{-kt}##, then ##x'(t) = -\frac g k e^{-kt} + C##. When you put ##t=0## in both sides you do not get ##C = v_0##.
 
Here are the equations in a latex format:

##\vec a = ge^{-kt}##
##\vec v = - \frac{g}{k} e^{-kt} + c_1##
##\vec r = \frac{g}{k^2} e^{-kt} + c_1t + c_2##

Using the initial condition ##r(0) = 0##, you should get something useful.

##- \frac{g}{k^2} = c_2##

The initial condition that ##v(0) = 0## also gives you some more information.

##\frac{g}{k} = c_1##

Then:

##\vec r = \frac{g}{k^2} e^{-kt} + \frac{g}{k} t - \frac{g}{k^2}##
 
  • Like
Likes 1 person
Oh, I see! So that's how to deal with the c's that wind up after integration. Excellent, thank you very much!
 
Question: A clock's minute hand has length 4 and its hour hand has length 3. What is the distance between the tips at the moment when it is increasing most rapidly?(Putnam Exam Question) Answer: Making assumption that both the hands moves at constant angular velocities, the answer is ## \sqrt{7} .## But don't you think this assumption is somewhat doubtful and wrong?

Similar threads

Replies
2
Views
2K
  • · Replies 1 ·
Replies
1
Views
1K
  • · Replies 2 ·
Replies
2
Views
1K
  • · Replies 7 ·
Replies
7
Views
2K
  • · Replies 2 ·
Replies
2
Views
1K
  • · Replies 1 ·
Replies
1
Views
1K
  • · Replies 7 ·
Replies
7
Views
2K
Replies
3
Views
2K
  • · Replies 3 ·
Replies
3
Views
2K
  • · Replies 3 ·
Replies
3
Views
2K